How to Choose Limits for Double Integrals?

Click For Summary
The discussion revolves around finding the correct limits for double integrals over a triangular region defined by the vertices (-1,0), (0,2), and (2,0). The user successfully calculated the integral using one method, obtaining the correct answer of 2 by integrating first with respect to x and then y. However, when attempting a second method that involved splitting the integral across the y-axis, the user arrived at an incorrect result of 8/3. The responses suggest that while the limits appear correct, the user may have made an error in their calculations, and further details are needed to identify the mistake. Clarifying the calculations is essential for understanding where the discrepancy lies.
applestrudle
Messages
64
Reaction score
0

Homework Statement



∫∫ydxdy over the triangle with vertices (-1,0), (0,2), (2,0)

Homework Equations




I did it like this and got the right answer:

∫dy ∫ydx

this first:
∫ydx from x = (y-2)/2 to x = 2-y

then ∫dy from y = 0 to y = 2

I got 2 which is correct


but when I tried another method, splitting them up across the y-axis i got 8/3:

I did I = A+B

A = ∫dx∫ydy

∫ydy from y = 0 to y = 2x+2

then I integrated from x = -1 to x =0 (I got 2/3)

B = ∫dx∫ydy

∫ydy from y =0 to y = -x+2

then I integrated wrt x from x = 0 to x = 2 (I got 2)

so the second answer was 8/3


Why was the second one wrong? I keep getting questions wrong because of the limits I choose! Could someone help me out please?




The Attempt at a Solution

 
Physics news on Phys.org
applestrudle said:

Homework Statement



∫∫ydxdy over the triangle with vertices (-1,0), (0,2), (2,0)

Homework Equations




I did it like this and got the right answer:

∫dy ∫ydx

this first:
∫ydx from x = (y-2)/2 to x = 2-y

then ∫dy from y = 0 to y = 2

I got 2 which is correct


but when I tried another method, splitting them up across the y-axis i got 8/3:

I did I = A+B

A = ∫dx∫ydy

∫ydy from y = 0 to y = 2x+2

then I integrated from x = -1 to x =0 (I got 2/3)

B = ∫dx∫ydy

∫ydy from y =0 to y = -x+2

then I integrated wrt x from x = 0 to x = 2 (I got 2)

so the second answer was 8/3


Why was the second one wrong? I keep getting questions wrong because of the limits I choose! Could someone help me out please?




The Attempt at a Solution


Your limits look to be correct with both methods. Without you showing your calculations instead of just giving us your answers, we can't tell where your mistake is.
 
Question: A clock's minute hand has length 4 and its hour hand has length 3. What is the distance between the tips at the moment when it is increasing most rapidly?(Putnam Exam Question) Answer: Making assumption that both the hands moves at constant angular velocities, the answer is ## \sqrt{7} .## But don't you think this assumption is somewhat doubtful and wrong?

Similar threads

Replies
4
Views
2K
Replies
2
Views
2K
  • · Replies 4 ·
Replies
4
Views
2K
  • · Replies 1 ·
Replies
1
Views
2K
  • · Replies 19 ·
Replies
19
Views
2K
Replies
2
Views
1K
  • · Replies 10 ·
Replies
10
Views
2K
Replies
24
Views
3K
  • · Replies 17 ·
Replies
17
Views
4K
  • · Replies 4 ·
Replies
4
Views
2K